• Skip to content

Free GMAT Test Questions

Welcome to our Question Bank!

You have not answered any question so far. There are 50 free practice questions in our database in total, which you can answer and will improve your skills.

You can answer all questions in a row (click on "All Questions") or only all questions of a particular section (click on that Section) or a single selected question (click on that Question).

All Questions

Quantitative reasoning - problem solving, quantitative reasoning - data sufficiency, verbal reasoning - critical reasoning, verbal reasoning - reading comprehension, verbal reasoning - sentence correction.

GMAT is a registered trademark of the Graduate Management Admission Council (GMAC), which is unaffiliated with and does not endorse this website.

Adaptive Testing Explained

Image

Wouldn't it be great to know the average GMAT score of the business school of your dreams? You've studied hard for the GMAT, but are you fully sure of what ...

Sentence Correction Secrets

Image

Robo-Reader Report

  • Skip to content

GMAT Prep Courses & Tutoring

Free GMAT Practice Questions

Given that:

4 m + n = 20 ; and

How many ordered pairs (m,n) exist in which m and n both are integers?

  • Menlo Coaching
  • Practice Questions
  • Problem Solving

Are you ready for the GMAT Focus Edition? Save 10% on our GMAT Prep Course, starting March 16.

Sample GMAT Problem Solving Questions

We’ve already covered why studying with official practice questions is the best way to prepare for the GMAT .  But even if you come up with the correct answer to an official problem, you still might not understand the underlying principles used to create that particular question, leaving yourself open to traps and pitfalls set by the test writers.  In the explanations below, I will use some of the core tenets of the Menlo Coaching GMAT curriculum to breakdown two official GMAT problem solving questions and provide important principles for correctly attacking this question type in the future. 

Multiple choice “problem solving” questions are, to most students, familiar, yet they generally do not approach them properly. To succeed on these questions, you obviously need the requisite knowledge related to the content area being tested—math skills related to arithmetic, algebra, etc. However, it is just as important to read carefully, leverage every hint, and choose the right strategy (backsolving, number picking, conceptual thinking, etc.) People think of multiple-choice problem solving questions as just plain math questions, but this GMAT sample question shows that they are much more than that. Take a look at the following questions, and check out our problem solving video below.

GMAT Problem Solving, Sample Question #1

Rates for having a manuscript typed at a certain typing service are $5 per page for the first time a page is typed and $3 per page each time a page is revised. If a certain manuscript has 100 pages, of which 40 were revised only once, 10 were revised twice, and the rest required no revisions, what was the total cost of having the manuscript typed?

GMAT Problem Solving, Sample Question #2

A certain airline’s fleet consisted of 60 type A planes at the beginning of 1980. At the end of each year, starting with 1980, the airline retired 3 of the type A planes and acquired 4 new type B planes. How many years did it take before the number of type A planes left in the airline’s fleet was less than 50 percent of the fleet?

Sample GMAT Questions by Topic

  • Data Sufficiency: Practice for the GMAT with Official Data Sufficiency Sample Questions
  • Data Insights: How to Approach Data Insights: Practice Questions and Explanations
  • Reading Comprehension: How GMAT Reading Comprehension Questions Mislead Test Takers: Practice Questions and Explanations
  • Critical Reasoning: How to Succeed Against Official GMAT Critical Reasoning Questions

Book a GMAT Strategy Call with Hailey

gmat problem solving pdf

Replies in 24 hours

[email protected]

✓ Get feedback on your profile as a test-taker

✓ Ask questions about strategy, timelines, or the GMAT itself

✓ Learn what you need to work on to improve your score

✓ Find out whether you’d be a fit with live classes or private tutoring

GMAT Prep Online Guides and Tips

2000+ gmat sample questions: practice every question type.

gmat problem solving pdf

Practice questions are an essential part of any GMAT prep. But how do you find the best GMAT sample questions?

In this article, I’ll address what to look for in GMAT practice questions, the pros and cons of both official and unofficial GMAT test questions, and tips on creating an effective GMAT study plan using these resources.

What to Look for in GMAT Sample Questions

There are a wide variety of GMAT sample questions out there. How do you know if you’re using quality practice ones? These are a few of the characteristics of good GMAT practice questions, whether they’re official or unofficial.

#1: Same Format and Question Types as the Real GMAT

Once you start taking GMAT practice tests, you’ll get a feel for how GMAT questions should look: how long they are, what they look like on the screen, the topics they tend to cover, and the style in which they’re written, for example. In every section, there will also be a mix of question types: the quant section always includes both data sufficiency and problem-solving questions, while the verbal section always includes a mix of sentence correction, reading comprehension, and critical reasoning questions.

The GMAT example questions you choose as you prepare for the test should be as similar as possible to real questions written by the Graduate Management Admissions Council (GMAC). The closer you can get to simulating real testing conditions, including everything from visuals to question content, the more comfortable you’ll feel on the day of the test. Also, answering GMAT practice questions that are similar to those you’ll encounter on exam day will help you gauge your progress and potential GMAT score range accurately.

Not sure how or what to study? Confused by how to improve your score in the shortest time possible? We've created the only Online GMAT Prep Program that identifies your strengths and weaknesses, customizes a study plan, coaches you through lessons and quizzes, and adapts your study plan as you improve.

We believe PrepScholar GMAT is the best GMAT prep program available , especially if you find it hard to organize your study schedule and don't want to spend a ton of money on the other companies' one-size-fits-all study plans.

     Improve Your GMAT Score by 60 Points, Guaranteed     

#2: Computerized (and Hopefully Adaptive)

The GMAT is a computerized adaptive test (CAT), which means it adapts to the test taker’s skill level in real time to determine both their score and the mix of questions they’re given. Ideally, GMAT sample questions should follow this format, so you can be best prepared for what you’ll see and experience on test day. Practice resources in CAT format will usually advertise that fact prominently. If they don’t, they’re likely not in the format of the official GMAT.

Obviously not every resource you use will be in CAT format—for example, if you’re using a book to do drills on specific question types. But a sizable portion of your prep should be done using practice questions in CAT format.

#3: Accurate Difficulty Levels

The GMAT always includes a mix of easy, medium and difficult questions. How many you receive of each will depend partly on the computerized adaptive test: You’ll start with ‘medium’ questions, and if you answer those correctly, you’ll be given more difficult questions on average; if you answer them incorrectly, you’ll get easier questions on average. However, every test taker will receive some questions at all three difficulty levels.

Your selection of practice GMAT questions should be at the same difficulty level as the real GMAT test questions. As you prepare, you’ll be able to tell if your sample GMAT questions seem too easy or too difficult on average. Practicing with either too-difficult or too-easy questions might give you an inaccurate picture of your probable ultimate GMAT score, and could also leave you underprepared for the real GMAT.

#4: Organized by Skill and/or Difficulty

You should target your weaknesses when completing GMAT practice questions. You may struggle more with data sufficiency questions than with problem-solving questions on the quant section, for example, or you may do well on most geometry questions but wrestle more with trigonometry and algebra. Alternatively, you may have mastered easy questions across the board and might need to focus on medium and/or difficult questions instead.

Resources that offer customizable sets (based on question type, skills tested, and difficulty level) of GMAT practice questions, or that separate the questions according to those categories, will help you study more effectively and in a more organized fashion than ones that aren’t labeled according to type or that are randomly compiled. The more specifically each question is categorized, the more effectively you will be able to form a study plan that hones in on your specific problem areas.

Computerized adaptive practice questions will help you prepare most effectively for the GMAT.

Official GMAT Practice Questions

The Graduate Management Admissions Council (GMAC) releases a variety of practice resources that use official GMAT sample questions, either written by the test makers themselves or adopted from retired exams. For each resource, I’ll go over what’s included, its strengths and weaknesses, and the best way to make use of it in your prep.

#1: GMATPrep Software

The official GMATPrep Software is the best resource for GMAT practice questions available. It contains two full-length simulated computerized adaptive tests . Everything is written by GMAC, so you know you’re getting the real thing. You can take the two practice tests as many times as you like, but you’ll likely repeat some of the same questions if you take each one more than once.

You can customize your sets of practice questions, choosing 1-15 questions of each type (critical reasoning, sentence correction, and reading comprehension, for verbal) and difficulty (easy, medium, or difficult). In addition to the questions on the two practice tests, there are 90 additional practice questions available on the software.

The software also includes a detailed breakdown of GMAT question types and strategies, as well as a review of skills you’ll need for the quant section.

  • It’s free for registered users of mba.com. Just create an account.
  • The practice questions are identical in length, format, style, content, and visuals to what you’ll see on the GMAT.
  • Answer explanations are step-by-step and in-depth, and you can go back and review or drill questions you previously got wrong as many times as you like.
  • Detailed performance reviews break down what you’re getting wrong so you can hone in on your weak spots, including specific question types and skill sets
  • Not much! The GMATPrep exams are great GMAT example tests. There aren’t many resources that can beat official questions under simulated testing conditions, and it’s free.
  • However, there are only two full-length practice tests, so you’ll likely need to supplement these practice questions with other resources.
  • Also, while the quant review is great, there isn’t a similar resource available on the software for the verbal section.

How to Use It:

  • It’s best to start off your GMAT prep by taking one of the two full-length simulated tests on the GMATPrep software. Since the practice questions and score report are so realistic, the results will serve as a good barometer of your starting point.
  • After you complete the rest of your prep with other practice questions, you should take the second GMATPrep practice test a few weeks before you take the GMAT. This will give you a good idea of where you stand before exam day.

#2: GMATPrep Question Pack 1

An official addition to the GMATPrep Software, the GMATPrep Question Pack contains 404 additional official questions.

You can create custom sets of practice questions of any length based on your needs. Each set of questions can be completed in either ‘exam mode’ (timed, under simulated test conditions) or ‘study mode’ (untimed). The Question Pack will cost you $29.99 to download.

  • This resource contains practice questions that aren’t available in any other official prep materials.
  • Your performance reviews for these practice questions will be integrated into your overall progress reports in the GMATPrep Software , allowing you to get an even more accurate picture of what your needs are in terms of prep.
  • There isn’t much of a downside to official questions as a supplement to your overall prep.
  • Once you’ve taken a few practice tests and figured out your weaknesses, create custom practice question sets that target your difficult spots (particular sections, question types, or question difficulty levels). These will serve as drills to beef up your skills in those areas.

GMAT customizable question banks allow you to work on your timing.

#3: The GMATPrep Exam Collection

These are official additions to the GMATPrep Software. You can only get them once you’ve downloaded the original free software.

Each Exam Pack contains 90 additional practice questions (30 quantitative, 45 verbal, and 15 integrated reasoning), as well as two full-length computerized adaptive GMATs. Each Exam Pack costs $49.99.

Want to improve your GMAT score by 60 points?

We have the industry's leading GMAT prep program. Built by Harvard, MIT, Stanford, and Wharton alumni and GMAT 99th percentile scorers, the program learns your strengths and weaknesses and customizes a curriculum so you get the most effective prep possible.

Try PrepScholar GMAT for 5 Days Risk-Free.

  • In-depth diagnostics will let you know how you’re doing in comparison to your peers on every subsection of the test , as well as how your pacing could improve on each question type.
  • Answer explanations offer step-by-step, detailed analyses of all practice questions.
  • Not much (official GMAC questions are always helpful), except that they’re a bit pricey.
  • These practice questions are a great first step after you take an initial diagnostic practice test, particularly if you’re still unsure of what exactly your weak spots are.
  • You can use the two full-length CAT GMATs to gauge your progress throughout your prep. Alongside the official GMATPrep Software, you’ll have four full-length tests, so you can space them out to regularly check up on how your score is improving.

#4: GMAT Write

GMAT Write is a fairly new tool released by the makers of the GMAT, meant to help you with the analytical writing assessment. It includes two unique sample prompts for the analytical writing assessment section.

GMAT Write will time your essays (30 minutes each, just like on the real exam) and score them in real time according to the GMAC rubric. It costs $29.99 to download.

  • The visuals and experience of writing the essay are highly realistic.
  • The sample questions are written by GMAC, so you know they’re reliable.
  • GMAT Write scores your essays according to the same criteria used on the real GMAT— ‘analyzes the issue,’ ‘supports ideas,’ ‘organizes a coherent idea,’ and ‘language control’ — in real time. It’s the only official GMAC tool that will score your sample essays.
  • There are only two unique exam prompts included, which isn’t necessarily a lot of practice for $29.99.
  • You won’t get any detailed feedback , just scores in each of the four categories used to assess GMAT essays.
  • Use it alongside other practice tests. Many other resources only have integrated reasoning, verbal and quantitative practice questions, so adding a prompt from GMAT Write to your practice test will help you more closely simulated the experience of the actual GMAT.

GMAT Write will help you prep for the Analytical Writing Assessment.

#5: The  GMAT Official Guide 2018

The GMAT Official Guide 2018  is a comprehensive resource for GMAT prep. Included in your purchase ($19.95) is access (both online and in print) to over 900 official practice questions, access to an accompanying site where you can customize sets of practice questions, and online videos with plenty of GMAT tips and strategies.

  • The questions are written by GMAC, so they’re high quality.  The online practice questions mimic the visuals of the real GMAT.
  • The practice questions are organized in order of difficulty.
  • The answer explanations and introduction sections (where key GMAT concepts get broken down) are too complex for non-advanced students. This is especially true for the quant and sentence correction questions.
  • The online question bank only allows you to save ten practice sessions at a time, so if you plan on doing more, you’ll need to delete them before starting a new session.
  • The online question bank is great for regular practice sessions. The question sets are customizable, and you can review them at any time.
  • The book itself is a good resource to start your practice with after using the GMATPrep software as an initial diagnostic tool.

#6: IR Prep Tool

The Integrated Reasoning (IR) Prep Tool was introduced in 2014 to aid students specifically in the integrated reasoning section.

The tool includes 48 integrated reasoning questions total: 10 graphics interpretation questions, 15 multi-source reasoning questions, 15 two-part analysis questions, and eight table analysis questions.

You can create customized practice question sets, focusing on all question types or just one. You can also set the difficulty level for each practice question set. Once downloaded, you have unlimited use of the prep tool for six months. The IR Prep Tool can be downloaded for $19.99.

  • A detailed ‘help’ section breaks down each integrated reasoning question type for you and gives tips on how to approach it.
  • A time management tool tracks your average time on each question type and difficulty level, allowing you to work specifically on your pacing.
  • The IR Prep Tool doesn’t give you a specific score estimation. Your official IR score will be between 1 and 8, but the IR Prep Tool gives you a score between 0 and 100 based on your performance and time management.
  • There aren’t many resources out there specifically aimed at the integrated reasoning section, a frequent problem area for students, so this is a goldmine for students who have trouble with it.
  • Use it to target and break down the integrated reasoning section into manageable chunks, particularly if you get overwhelmed by the fact that each question has multiple components.

#7: GMAT Focus Quantitative Diagnostic Tool

The GMAT Focus Quantitative Diagnostic Tool consists of a 24-question quantitative test (12 data sufficiency questions and 12 problem-solving questions ) that uses real questions from retired exams. It’s computer adaptive and follows the style and format of the actual GMAT quant section.

You can start and stop the test manually, but you have 45 minutes total to complete the practice questions.

Want to Identify YOUR GMAT Strengths and Weaknesses?

Our proprietary GMAT Diagnostic Assessment creates a customized study plan for you that takes you from registration all the way to test day! It is included with every account and proven to significantly maximize your score .

Get your personalized assessment as part of your 5 day risk-free trial now:

Get Your Free GMAT Diagnostic Assessment Here

When you complete the test, the Focus Tool provides detailed answer explanations and an assessment of your strengths and weaknesses in terms of quant concepts.

Four unique tests are available. One test costs $29.99, and a three-test package costs $79.99.

  • The GMAT Paper Tests are the only other GMAT practice resources that use real retired GMAT questions, making the Focus Tool the only computerized adaptive resource to do so. So you get the best of both worlds with this one: real practice questions from previous GMATs and simulated testing conditions.
  • The detailed assessment of your performance, including a possible score range (more accurate than an exact score, since no one practice test will give you a perfect prediction of your ultimate GMAT score) and a breakdown of your performance by question difficulty, will tell you exactly which quant skills you need to work on before the GMAT.
  • It’s a bit pricey for just 24 questions each.
  • It’s not tied into the other GMAT prep resources, so your results won’t be integrated into your GMATPrep performance reports.
  • After you complete one of the GMATPrep exams, this tool will help you hone in on the quant section . You’ll get an overall idea of your strengths and weaknesses in quant from the GMATPrep Software, but this tool will help you get much more specific about which skills to target in your quant prep.

The Quantitative Focus Diagnostic Tool will help you hone in on the intricacies of the GMAT quant section.

#8: GMAT Paper Tests

The GMAT Paper Tests are real retired GMAT tests , written by GMAC. Each set (I, II, and III) contains three official GMAT tests, answer sheets, and a guide for converting your raw score into a scaled score. They’re available in downloadable PDF form. Each set costs $29.99.

  • Most of the questions in these tests aren’t available in any of the other official GMAT practice tests or resources, so they’re especially good if you want some extra practice. This is a big plus: It’s somewhat difficult to find GMAC-authored questions that don’t overlap with other resources.
  • Official questions are always a big positive in terms of GMAT prep!
  • They’re on paper! You’ll have to score them yourself using the answer sheet, and it’s not computerized (obviously) or adaptive, so it doesn’t simulate any of the testing conditions.
  • No answer explanations are included, just the answers themselves.
  • You may want to order these if you’re a test-taker with a disability that may require an accommodation and using paper tests at home is easier for you during prep time.
  • The GMAT Paper Tests might also serve you well if you’ve used all the other official sample GMAT questions and you want some additional prep. However, don’t rely on them to give you much info on your likely final GMAT score, since they aren’t in the official GMAT format. Use them to work through practice questions in your prep sessions instead.

Best Unofficial GMAT Practice Questions

Official GMAT questions are great, but they aren’t the only GMAT prep materials out there. There are also a variety of free and paid resources that offer quality GMAT practice questions. Here, I’ve included practice tests, books, and online question banks. For each unofficial resource, I’ll address how you can access it, its pros and cons, and the best way to incorporate it into your GMAT prep.

#1: Veritas Prep

You’ll need to create an account to access Veritas Prep’s GMAT Question Bank, but once you do, it’s completely free.

The Question Bank offers hundreds of realistic sample GMAT questions: quant, verbal, and integrated reasoning. It includes example questions of every type within those categories. Before you start your practice session, you can choose which kinds of questions you want to see. With one account, you’ll only see each practice question once.

Veritas Prep doesn’t give you an official estimated score, but it does give you an overall percentile , which measures how you did on a particular question set compared to other students who answered the same practice questions.

The Veritas Prep Question Bank is ideal for regular practice throughout your prep, particularly if you have varying amounts of time (you can do a short or lengthy session) and want to drill certain skill sets or question types . The questions from the bank aren’t in CAT format, but you can access one free computerized adaptive test through Veritas and six paid ones. Veritas Prep questions are some of the closest you’ll find to official GMAC-authored GMAT example questions.

Kaplan has a wide variety of GMAT resources, including in-person and online prep courses, but its GMAT Premier 2018  is its flagship GMAT prep tool. Your purchase includes access to over 1,000 GMAT practice questions, six full-length practice tests (one is in the book, and five are computerized adaptive tests available online), video tutorials, an online question bank with 200 GMAT sample questions and customizable quizzes, and a mobile app that allows you to study on the go.

GMAT Premier also includes a special section of forty advanced quant practice questions and in-depth answer explanations, which can help if you’re looking for a challenge in the quant department. The Kaplan practice questions are a bit harder than the ones in Princeton Review, but the verbal questions in particular still leave something to be desired in terms of difficulty level. The sentence correction questions in particular are easier than real GMAT questions. The Kaplan quant questions fare a bit better in terms of GMAT comparability.

Like Princeton Review, Kaplan is a good resource if you’re looking for a fairly solid overview and breakdown of the exam to start your prep (after you take your first diagnostic GMATPrep test). It’s not going to be as helpful if you’re looking for a comprehensive guide to the GMAT or a massive boost in your score.

#3: Manhattan Prep

With an account, you can take one full-length computerized adaptive Manhattan Prep test for free. Six additional ones are available for $49.99, which gives you one year of unlimited access.

You can take the Manhattan Prep practice GMATs timed or untimed, and you can take complete or partial practice exams depending on your time limit during prep. After you take all six, you can reset the questions to take more. Each test includes an in-depth assessment of your performance that analyzes your strengths, weaknesses, timing, and the difficulty levels you’ve mastered.

Manhattan Prep questions are mostly quite realistic in content and difficulty level. Though the visuals aren’t perfect, they are not too far from what you’ll see on the real exam.

I’d also like to point out one of Manhattan Prep’s best resources for students looking to challenge themselves in the quant section: Their GMAT Advanced Quant book includes strategies for tackling the trickiest quant questions and over 150 realistic practice questions. This is a great resource if you’re already doing well in the quant section and looking to bump yourself up to the next level.

#4: PowerScore

PowerScore’s main claim to fame is its trilogy of GMAT Verbal Bibles: Critical Reasoning, Sentence Correction, and Reading Comprehension, along with the PowerScore Verbal Bible , which covers the entire section. The books are available for $24.99 each or $69.99 as a trilogy.

PowerScore does an excellent job of breaking down each question type within the verbal section (a great resource for non-native English speakers and anyone who struggles with verbal questions), and the sample GMAT questions included in each book are consistent with the length, difficulty, and content of those on the GMAT. Though of course the practice questions here aren’t computerized or adaptive, each book does include access to a companion website that includes extra drills and practice questions.

The main downside to PowerScore is that, while the books offer great strategies and explanations of various question types, there aren’t a substantial number of actual practice questions . The Verbal Bible , for example, contains 31 critical reasoning questions, 77 sentence correction questions, and only eight reading comprehension questions. So while these resources are great reads for test-takers who are looking for in-depth answer explanations, they’re not fantastic for substantial extra practice or regular prep sessions.

The PowerScore Verbal Bibles have excellent grammar guides to help you with sentence correction questions in particular.

#5: 800 Score Test

Providing a username and password will get you one full-length test on 800 Score Test for free. You can download five more for $39.95, which includes a comprehensive quant and verbal review, ten sample essay questions, access to online help, and in-depth video explanations of practice questions.

The visuals at 800 Score Test aren’t much like what you’ll see on the GMAT, but the practice questions themselves are high quality, particularly in the quant section. The verbal practice questions veer a little ‘off’ and aren’t as close to the real GMAT verbal section as the quant questions are, so if you’re especially looking to focus on verbal practice questions, this might not be your best bet.

The 800 Score tests are in CAT format, though, so even though the visuals are a bit lacking, they’re not the worst resource if you only want practice questions that simulate real testing conditions. Their Test Pacer tool lets you know what question you should be on to finish in time, which is very helpful for students who have trouble with consistent pacing.

#6: Princeton Review

Princeton Review’s Cracking the GMAT, available for $21.99 on Kindle, offers a breakdown of all GMAT sections (including integrated reasoning and the analytical writing assessment), along with six full-length computerized adaptive practice exams, over 180 practice questions organized by difficulty level, and drills for each test section. Cracking the GMAT Premier is $7 more and comes with more online resources, like extra practice tests, study strategies, and video tutorials.

Princeton Review offers a decent overview of each of the GMAT sections, though it only offers the grammar and math fundamentals, not anything in the way of more advanced concepts. Indeed, where it misses the mark is mainly in the difficulty of the practice questions: Nearly none of them are as difficult as actual GMAT questions. This is especially true in the critical reasoning and reading comprehension sections. So while Cracking the GMAT may help beginners who aren’t familiar with GMAT basics (especially to start off, since it’s not an intimidating text), it’s not enough for test-takers looking to break 700 or even 650.

#7: GMAT Club

Beyond its helpful GMAT forums, GMAT Club offers a variety of useful GMAT practice materials, including practice tests and question banks.

A few practice tests and question banks are free (no account required), but access to all of their customizable quizzes, bonus questions (over 1,572) and full-length tests will run you at least $79.99 for three months of use.

GMAT Club tests aren’t exactly in CAT format, as they don’t adapt to your performance as you go along, but they do follow CAT ‘logic’ and include a mixture of difficulty levels and question types that is consistent with what you’ll usually encounter in a computerized adaptive test. The quality of the visuals and practice question content is high, with practice questions closely resembling GMAC ones.

Difficult quant questions are one of GMAT Club’s specialties, which is important as many of the other top practice resources (Princeton Review and Kaplan, for example) trend a little easier: They claim that hundreds of their quant questions are at the 700+ scoring level. So if you’re looking to wow your prospective schools with your quant score, this might be a great resource.

#8: Manhattan Review

You have several options for free practice questions and other GMAT prep materials with Manhattan Review. With an account, you can access a fairly high-quality quantitative question bank in PDF ebook form, along with a vocabulary list (good for non-native English speakers) and another free ebook, GMAT in a Nutshell , that serves as an overall guide to the test and top strategies.

The Sentence Correction Guide, downloadable for free, boasts a solid grammar review of the concepts that are more often tested in sentence correction questions, but the sentence correction practice questions themselves aren’t all quite as difficult or complex as the real ones on the GMAT.

Manhattan Review also offers a full-length free practice test, which you can access with an account. It’s a computerized adaptive test. The questions here are good for extra practice, but the difficulty levels and visuals aren’t always consistent with the actual GMAT.

Vocabulary practice is an integral part of GMAT prep for non-native English speakers.

3 Top Tips for Creating a GMAT Study Plan

Once you’ve selected some practice GMAT questions, it’s time to set up a study plan. It’s important not just to find the best prep materials for you, but to use them to your advantage. Here are a few tips for using your prep materials as effectively as possible:

#1: Use the Two Official GMAC Tests on the GMATPrep Software First and Last

You should start your prep with one of the two full-length GMATPrep Software tests. Since they’re computerized adaptive tests with accurate visuals and questions written by GMAC, they’re the best possible indicator of your ultimate GMAT score. The first practice test will give you a good idea of where you stand and how to begin your prep. Unless you purchase more official GMAC practice tests, take the second free test from the software a few weeks before your exam date. This will give you a good idea of your score at the end of it all, and will let you know what to brush up on last-minute before the big day.

#2: Use Different Materials to Build Different Skills

Tailor the practice GMAT materials you select to your particular needs. For example, if you’re strong in quant but weaker in verbal (or a non-native English speaker), you might use Manhattan Prep, the GMAT Focus Quantitative Diagnostic Tool, or GMAT Club practice questions for your quant prep, since they all feature more advanced quant questions. You might start out more slowly in verbal, completing grammar reviews and drills from one of the PowerScore Bibles before delving more into practice questions. Mix and match resources according to your personal GMAT goals.

#3: Note Differences Between the Conditions of Your Practice and the Conditions of the Real GMAT

Not all of your GMAT practice questions need to be (or should be) in computerized adaptive form. Sometimes you’ll need to take your time to work through a particular question, complete a drill, or review concepts. But when you do take a full-length GMAT practice test, stick to the real testing conditions as much as you can: Use double-sided scratch paper, don’t use a calculator on the quant section, take the right amount of time for each section, and take eight-minute breaks between the integrated reasoning and quant sections and before the verbal section.

Some GMAT practice materials allow you to manually time yourself, take longer or unlimited breaks, or stop and start the test. Be mindful of these differences so you don’t learn to rely on conditions (such as a longer break) that won’t exist on the day of your exam.

whats-next-1462747_1280

What’s Next?

Looking for free full-length practice tests in addition to the best practice questions? You can find a list in our guide to the best free GMAT practice tests .

Our complete collection of GMAT practice tests provides a more comprehensive overview, in addition to reviews, of all available GMAT practice tests.

Finally our guide to how long to study for the GMAT will help you set up a time-effective study plan.

Was this helpful? Sign up for FREE GMAT and MBA guides!

Share this:.

  • Click to share on Twitter (Opens in new window)
  • Click to share on Facebook (Opens in new window)
  • Click to share on Google+ (Opens in new window)

gmat problem solving pdf

Author: Laura Dorwart

Laura Dorwart is a Ph.D. student at UC San Diego. She has taught and tutored hundreds of students in standardized testing, literature, and writing. View all posts by Laura Dorwart

MBA Watch Logo

Protected: GMAT Problem Solving—Be Flexible in Your Approach (and Know What You Need to Know)!

  • Share on Facebook
  • Share on Twitter
  • Share on LinkedIn
  • Share on WhatsApp
  • Share on Reddit

This content is password protected. To view it please enter your password below:

Questions about this article? Email us or leave a comment below.

  • Stay Informed. Sign Up! Login Logout Search for:
  • What Matters? And What More? 50 Successful Essays To The GSB & HBS
  • Specialized Masters Program Directory Business Analytics Hub MBA Admissions Consultant Directory Online MBA Hub Home Assess My MBA Odds
  • Acceptance Rates & Yield At The Top 100 U.S. MBA Programs (21,566 views)
  • GRE Scores & Submission Rates At The Top 50 U.S. MBA Programs (9,257 views)
  • The Top 10 MBA Concentrations With The Best ROI (8,678 views)
  • How U.S. News Ranks The Top U.S. B-Schools By MBA Specialization (4,292 views)
  • Poets&Quants 2023-2024 MBA Ranking: Stanford’s Triumphant Return To The Top (4,247 views)

gmat problem solving pdf

Our Partner Sites: Poets&Quants for Execs | Poets&Quants for Undergrads | Tipping the Scales | We See Genius

gmat problem solving pdf

  • Exam Prep >
  • Prepare for Business School >
  • Business School & Careers >
  • Explore Programs >
  • Connect with Schools >
  • How to Apply >
  • Help Center >

Every journey needs a plan. Use our Career Guide to get where you want to be. 

  • About the Exam
  • Register for the Exam
  • Plan for Exam Day
  • Prep for the Exam
  • About the Executive Assessment
  • Register for the Executive Assessment
  • Plan for Assessment Day
  • Prepare for the Assessment
  • NMAT by GMAC
  • Shop GMAT Focus Official Prep
  • About GMAT Focus Official Prep
  • Prep Strategies
  • Personalized Prep Plan
  • GMAT Focus Mini Quiz
  • Executive Assessment Exam Prep
  • NMAT by GMAC Exam Prep

Prepare For Business School

  • Business Fundamentals
  • Skills Insight

Business School & Careers

  • Why Business School
  • Student Experience
  • Business Internships
  • B-School Go
  • Quiz: Are You Leadership Material?
  • MBA Return on Investment (ROI) Calculator
  • Estimate Your Salary
  • Success Stories
  • Diversity and Inclusion
  • Women in Business

Explore Programs

  • Top Business School Programs
  • Quiz: Which Post Graduate Program is Right for You?
  • Quiz: Find the Best Program for Your Personality
  • Business School Rankings
  • Business Master's Programs
  • MBA Programs
  • Study Destinations
  • Find Programs Near Me
  • Find MBA Programs
  • Find Master's Programs
  • Find Executive Programs
  • Find Online Programs

Connect with Schools

  • About GradSelect
  • Create a GradSelect Profile
  • Prep Yourself for B-School
  • Quiz: Can You Network Like An MBA?
  • Events Calendar
  • School Events
  • GMAC Tours Events
  • In-Person Events
  • Online Events

How to Apply

  • Apply to Programs
  • The Value of Assessments
  • Admissions Essays
  • Letters of Recommendation
  • Admissions Interviews
  • Scholarships and Financing
  • Quiz: What's Your Ideal Learning Style?

Help Center

  • Register for the GMAT
  • Create Account

Sample Questions

Quick links.

Before you register  for the GMAT Focus Edition, get a preview of the types of questions you’ll encounter on the exam. 

Remember: You can learn more about section and question type at Exam Content .

Sample Questions by Section

Quantitative reasoning.

Problem Solving Question Directions: Solve the problem and indicate the best of the answer choices given. Question: If a certain wheel turns at a constant rate of x revolutions per minute, how many revolutions will the wheel make in k seconds? (A) 60 kx (B)  kx (C) x ÷ k (D) x ÷(60 k ) (E) kx ÷60 Answer:  (E)

Verbal Reasoning

Reading Comprehension Question Directions: The questions in this group are based on the content of a passage. After reading the passage, choose the best answer to each question. Answer all questions following the passage on the basis of what is stated or implied in the passage. Question: Schools expect textbooks to be a valuable source of information for students. My research suggests, however, that textbooks that address the place of Native Americans within the history of the United States distort history to suit a particular cultural value system. In some textbooks, for example, settlers are pictured as more humane, complex, skillful, and wise than Native Americans. In essence, textbooks stereotype and depreciate the numerous Native American cultures while reinforcing the attitude that the European conquest of the New World denotes the superiority of European cultures. Although textbooks evaluate Native American architecture, political systems, and homemaking, I contend that they do it from an ethnocentric, European perspective without recognizing that other perspectives are possible. One argument against my contention asserts that, by nature, textbooks are culturally biased and that I am simply underestimating children's ability to see through these biases. Some researchers even claim that by the time students are in high school, they know they cannot take textbooks literally. Yet substantial evidence exists to the contrary. Two researchers, for example, have conducted studies that suggest that children's attitudes about particular cultures are strongly influenced by the textbooks used in schools. Given this, an ongoing, careful review of how school textbooks depict Native Americans is certainly warranted. Which of the following would most logically be the topic of the paragraph immediately following the passage? (A) Specific ways to evaluate the biases of United States history textbooks (B) The centrality of the teacher's role in United States history courses (C) Nontraditional methods of teaching United States history (D) The contributions of European immigrants to the development of the United States (E) Ways in which parents influence children's political attitudes Answer:  (A)

Critical Reasoning Question Directions: For this question, select the best of the answer choices given. Question: Mall Owner: Our mall’s occupancy rate is so low that we are barely making a profit. We cannot raise rents because of an unacceptably high risk of losing established tenants. On the other hand, a mall that is fully occupied costs about as much to run as one in which a rental space here and a rental space there stands empty. Clearly, therefore, to increase profits we must sign up new tenants. Which of the following, if true, most seriously weakens the argument? (A) The mall’s operating costs could be cut by consolidating currently rented spaces in such a way that an entire wing of the mall could be closed up. (B) The mall is located in a geographic area in which costs incurred for air-conditioning in the hot summers exceed those incurred for heating in the mild winters by a wider margin. (C) The mall’s occupancy rate, though relatively low, has been relatively stable for several years. (D) The mall lost tenants as a result of each of the two major rent increases that have occurred there. (E) None of the mall’s established tenants is likely to need additional floor space there in the foreseeable future. Answer:  (A)

Data Insights

Data Sufficiency Question Directions: This data sufficiency problem consists of a question and two statements, labeled (1) and (2), in which certain data are given. You have to decide whether the data given in the statements are sufficient for answering the question. Using the data given in the statements, plus your knowledge of mathematics and everyday facts (such as the number of days in July or the meaning of the word counterclockwise), you must indicate whether:

  • Statement (1) ALONE is sufficient, but statement (2) alone is not sufficient to answer the question asked.
  • Statement (2) ALONE is sufficient, but statement (1) alone is not sufficient to answer the question asked.
  • BOTH statements (1) and (2) TOGETHER are sufficient to answer the question asked, but NEITHER statement ALONE is sufficient to answer the question asked.
  • EACH statement ALONE is sufficient to answer the question asked.
  • Statements (1) and (2) TOGETHER are NOT sufficient to answer the question asked, and additional data specific to the problem are needed.

Question: What is the monthly rent for a certain apartment? (1) The monthly rent per person for 4 people to share the rent for the apartment is $375. (2) The monthly rent per person for 4 people to share the rent of the apartment is $125 less than the monthly rent per person for 3 people to share the rent. (A) Statement (1) ALONE is sufficient, but statement (2) alone is not sufficient. (B) Statement (2) ALONE is sufficient, but statement (1) alone is not sufficient. (C) BOTH statements TOGETHER are sufficient, but NEITHER statement ALONE is sufficient. (D) EACH statement ALONE is sufficient. (E) Statements (1) and (2) TOGETHER are NOT sufficient. Answer: (D) For the other four question types, click for samples of each question type to open up in an interactive pop-up window. Please note: these sample questions are built to simulate the actual test interface, and therefore, are not optimized for mobile devices.

  • Multi-Source Reasoning
  • Table Analysis
  • Graphics Interpretation
  • Two-Part Analysis

Study Materials for 2023 CFA®, FRM®, Actuarial, GMAT® and EA® Exams

  • Payment Plans
  • Product List
  • Partnerships

gmat problem solving pdf

  • Help & Feedback
  • Register/Log In
  • Study Packages
  • Levels I, II & III Lifetime Package
  • Video Lessons
  • Study Notes
  • Practice Questions
  • Levels II & III Lifetime Package
  • About the Exam
  • About your Instructor
  • Part I Study Packages
  • Part I & Part II Lifetime Package
  • Part II Study Packages
  • Exams P & FM Lifetime Package
  • Quantitative Questions
  • Verbal Questions
  • Data Insight Questions
  • Live Tutoring
  • About your Instructors
  • EA Practice Questions
  • Data Sufficiency Questions
  • Integrated Reasoning Questions

Slide

GMAT® Practice Questions

Get access to 1,000 exam-style questions.

gmat-qbank

What is the GMAT Quantitative Reasoning Questions Exam Structure?

The quantitative reasoning section tests your numerical literacy and mathematical abilities, such as solving quantitative problems and interpreting graphical data. It consists of 31 multiple choice questions, which should take  62 minutes (2 minutes per question) to be completed.

The quantitative reasoning section of questions consists of data sufficiency and problem-solving questions.

1. Data Sufficiency Questions Each Data sufficiency question is made up of a question and two statements. To answer a question, one should first identify the statement that provides information relevant to the question, and then eliminate all the other possible answers by using math knowledge and other everyday facts. There will be 14 to 15 questions on data sufficiency in each quantitative section.

  2. Problem-solving Questions Each Problem-solving question includes a question section and five possible answers to choose from. The questions are designed to assess your use of logic and analytical reasoning to answer quantitative problems.

gmat problem solving pdf

What are the Skills and Strategies to Excel in Quantitative Reasoning Questions?

Math skills.

Quantitative Reasoning questions call for the application of math knowledge to solving problems. Such mathematical skills include:

  • Basic arithmetic including fractions, integers, powers and roots, and statistics and probability.
  • Algebraic topics such as variables, functions, and solving equations.
  • Word problems such as algebraic and geometric principles and blending arithmetic used to solve problems.
  • Geometry, particularly geometrical objects such as triangles, circles, quadrilaterals, solids, cylinders, and coordinate geometry.

Essential Strategies in Problem Solving Questions

  • Always check your onscreen timer. Apply caution while solving questions but don’t waste too much time verifying answers; strive to complete a given section.
  • If you find a question difficult or time-consuming, try to eliminate answer choices that might be outrightly wrong and select your best choice from the remaining ones.
  • Study each question in depth to ascertain what is being asked. For example, you may need to come up with some equations.
  • Solve the questions by writing to limit errors. An erasable tablet will be provided at the test center.
  • For the data sufficiency questions, ascertain whether the question requires only one value or a range of values. Moreover, avoid unnecessary assumptions about geometrical figures, as they are not usually drawn to scale.
  • Go through the answer choices before answering a question to get the gist of what kind of answer choice structure you are required to present. Moreover, some questions require simple approximation presentations, which may require little mental activity rather than long computations.

Questions Answered by our Users

Satisfied customers, preparation platform by review websites, some free exam-style gmat practice questions offered by analystprep, gmat quantitative problems, division & factoring.

When a number \(b\) is divided by \(a\), the remainder is 12. Given that \(\frac{b}{a}=7.15\), what is the value of a?

The correct answer is: C)

The result 7.15 due to division gives us the quotient 7 and the remainder 0.15. Therefore, \(\frac{12}{a} =0.15\) implying that \(a=\frac{12}{0.15}=80\).

Descriptive Statistics

Dre has seven French tests this semester that is each scored out of a maximum of 100 points. If his average score on the first five tests was 87, what is the minimum score he could get on the sixth test and still be able to maintain his 87 average through all seven tests?

The correct answer is: A)

If the average score on Dre’s first five tests is 87, the sum of the first 5 scores is \(5 × 87 = 435\).

To maintain a score of 87 for all 7 tests, the total score required would be \(7 × 87 = 609\).

The maximum score Dre could get on the seventh test would be 100, so \(609 – 435 – 100 = 74\) would be the minimum score he could get on test 6 and still maintain his 87 point score average.

Linear Algebra

Scott began losing weight at a constant rate five months ago and today weighs 25 pounds less than he weighed then. Concurrently, his brother Karl has also been losing weight at three times the rate of Scott. If Karl weighed the same weight two months ago that Scott is today, and the two brothers weigh a combined 450 pounds today, what was Scott’s weight five months ago?

So, Scott’s weight today \(= s−25\).

Karl’s weight loss rate is three times that of Scott, i.e., 15 pounds per month. So, Karl has lost a total of \(15×5=75\) pounds over 5 months.

Karl’s weight today = k−75.

Given, their combined weight today is 450 pounds:

$$\begin{align*}s−25+k−75&=450\\ s+k&=550\end{align*}$$

From the problem, we know that Karl weighed the same as Scott does today 2 months ago. Thus, Karl’s weight 2 months ago was \(k−3(15)=k−45\), because he loses 15 pounds each month.

$$\begin{align*}s−25&=k−45\\ s−k&=-20\end{align*}$$

Now, plugging (2) into (1):

$$\begin{align*}k-20+k&=550\\ 2k-20&=550\\ 2k&=570\\ k&=285\end{align*}$$ And from (2): $$\begin{align*}s&=550-285\\ s&=265\end{align*}$$

Basic Quadratics

If \(y=x^2 +mx + 48\) has x-intercepts (a, 0) and (b, 0), where a and b are integers, what is the least possible value of m?

The correct answer is: D)

We know that ab=48 and a+b=m, so we just need to find all the possible combinations of a and b. We have the possible sets (1, 48), (2, 24), (3, 16), (4, 12), (6, 8), and their negative counterparts.

To find the value of m, we need to find the lowest sum of each pair of numbers.

It would be (-1, -48) whose sum is -49.

Functions & Symbols

For any two integers m and n, range(m, n) denotes the difference in the values of m and n. For example, range(3, 6) = 3. For the integers c and d, what is the value of range(c, d)?

(1) range(c, 6) = 20

(2) range(d, 15) = 5

A) Statement (1) ALONE is sufficient but statement (2) ALONE is not sufficient.

B) Statement (2) ALONE is sufficient but statement (1) ALONE is not sufficient.

C) BOTH statements TOGETHER are sufficient, but NEITHER statement ALONE is sufficient.

D) EACH statement ALONE is sufficient.

E) Statements (1) and (2) TOGETHER are not sufficient.

The correct answer is: E)

(1) This statement allows for two possible values of c, 4 or 8 and provides no information about d; NOT sufficient.

(2) This statement allows for two possible values of d, 10 or 20 and provides no information about c; NOT sufficient.

(Together) There are still multiple possible differences for range(c, d); NOT sufficient.

The correct answer is E; both statements together are still not sufficient.

A rectangular tennis court is divided into six sections with a net splitting the court into two halves at the midpoint of its length. Each half has three equal sections consisting of a single large section and two equal smaller sections exactly half the size of the large section. If the length of the entire court is 24 meters, and the area of one of the smaller sections is 24 square meters, what is the width of the entire court?

A) 4 meters.

B) 6 meters.

C) 8 meters.

D) 12 meters.

E) 24 meters.

If the area of two smaller sections of one half of the court is equal to half of that half of the court’s area, then \(24 = \frac{1}{2} × \frac{1}{2}lw\)

Since the length of half of the court is 12 meters, solve for \(w\) using the equation as \(24 = \frac{1}{2} × \frac{1}{2}(12)w\)

Multiply the full equation by 4 to find that \(96 = 12w\) and divide by 12 to find that \(8 = w\).

Rates and Work

Operating at the same time at their respective constant rates, Misha and Petro can repair 100 widgets in \(q\) hours. Working by himself at his constant rate, Misha repairs 100 widgets in \(w\) hours. In terms of \(q\) and \(w\), how many hours does it take Petro, working alone at his constant rate, to repair 100 widgets?

A) \(\frac {qw}{w-q}\).

B) \(\frac {qw}{q-w}\).

C) \(\frac {q}{w+q}\).

D) \(\frac {w}{w-q}\).

E) \(\frac {q}{q-w}\).

Use the combined time formula where combined time = \(\frac{time A × time B}{time A + time B}\), where time for Misha = \(w\), combined time = \(q\) and time for Petro = \(x\).

Therefore, \( q = \frac{xw}{x + w}\).

Multiply the equation by \(x + w\) to find \(qx + qw = xw\).

Subtract by \(qx\) and factor \(x\) to find that \(qw = x(w – q)\).

Divide by \(w – q\) to find that \(\frac{qw}{w – q} = x\).

GMAT ® Focus Prep Packages

Gmat focus complete course.

  • Conceptual Video Lessons
  • Quantitative Practice Problems
  • Verbal Practice Problems
  • Data Insight Practice Problems
  • Customizable Quizzes
  • Performance Tracking Tools

Testimonials

Stefan Maisner is without a doubt the best tutor I could have possibly found. Stefan knew the official questions like the back of his hand and he knew all of the shortcuts. Being that i have a background in communications, the quant section had been a pain point for months. Stefan also had a background in communications and he was able to teach me in the way that I understood the math concepts that I needed to grasp, and showed me how to think logically and critically about the problems that didn’t require you to do the math.

Courtney M.

I prepared for the GMAT under Stefan. It was super easy to use the online platform and was very convenient to save notes this way. Stefan is extremely knowledgable on the testing material and really cares about his students.

As an international student I did not have much standardized test taking experience and never had a tutor before. However, Stefan really helped me improve my core skills and also helped me develop crucial test taking strategies. I was able to see improvements after every single tutoring session with him. Overall, he helped me improve my GMAT score by 90 points (from 650 in diagnostic test to 740 on the actual test), which was way above what I thought was possible.

I had an amazing experience with Stefan M for my GMAT prep. When I started the process I was scoring in the low 600s, but by the end, I managed to get my score above a 700! Stefan was also great at helping me navigate the add uncertainties caused by the Covid-19 pandemic. I could not recommend him more enough!

I can definitively say that if I did not have Stefan to help me that there would be no way that I could’ve gotten the final score I did. His intimate knowledge of how the test works, the strategy required for each section, and his mastery of the content of the questions, puts him in the top echelons of GMAT tutors. I could not recommend him anymore to any student looking to take the GMAT.

Sebastian B.

Stefan was awesome! He brought up my verbal from the 40th to the 76th percentile. Very affordable, very high quality. I went from a 650 to a 690 in a very tight timeframe.

I had already taken the online prep course from another provider and had a score in the high 600s. I then worked with Stefan to prepare for my re-take of the GMAT. I was able to increase my Verbal score from 35 to 40 and finally get over the summit of 700 (720: Q48: V40.) I would like to thank him for all his help!

My lessons with Stefan started almost right away and the approach he taught me was much easier to understand than what was presented in the usual GMAT guide books (several of which I had read over multiple times, and failed to implement successfully). I took the test a second time and obtained a score of 740, which was my target.

Harshita V.

Stefan was awesome! He brought up my Verbal from the 40th to 76th percentile. I would definitely use him again. Very affordable, very high quality. I went from a 650 to a 690 in a very tight timeframe.

gmat problem solving pdf

Hit enter to search or ESC to close

gmat problem solving pdf

  • Examen P – Probabilités
  • Partenariats
  • CFA ® Niveau 1
  • CFA ® Niveau 2
  • CFA ® Niveau 3
  • FRM Partie 1
  • FRM Partie 2

Recently viewed courses

Recently viewed.

Find Your Dream School

This site uses various technologies, as described in our Privacy Policy, for personalization, measuring website use/performance, and targeted advertising, which may include storing and sharing information about your site visit with third parties. By continuing to use this website you consent to our Privacy Policy and Terms of Use .

   COVID-19 Update: To help students through this crisis, The Princeton Review will continue our "Enroll with Confidence" refund policies. For full details, please click here.

GMAT Sample Questions

Hidden laptop with hands typing, displaying a test diagram above the keyboard

Want a preview of the question types you'll face on the GMAT? Try your hand at the GMAT practice questions below. Then, check your answers against our in-depth explanations to see how you did.

We pulled these GMAT sample questions from our book Cracking the GMAT and from our test prep course materials. For more GMAT practice, take a full-length practice test with us held under the same testing conditions as the real thing. Find out how you'd score, and get  a personalized score report from us that shows your strengths and weaknesses.

  • GMAT Verbal Questions 
  • GMAT Math Questions
  • GMAT Integrated Reasoning Questions 
  • Essay Prompt 

Below you'll find sample GMAT questions covering the three question types you'll encounter on the Verbal section: Sentence Correction , Critical Reasoning, and Reading Comprehension.

GMAT Sentence Correction Questions

1. In order to better differentiate its product from generic brands, the cereal company first hired a marketing firm that specializes in creating campaigns to build brand awareness and then retools its factory to produce a variety of different shapes of cereal. (A) then retools its factory to produce a variety of different shapes of cereal (B) retools its factory to produce a variety of different shapes of cereal (C) then retooled its factory to produce a variety of different shapes of cereal (D) then will retool its factory to produce a variety of different shapes of cereal (E) then produces a variety of different shapes of cereal through retooling its factory

Answer: (C) The actions of the cereal company are not in parallel form. First the company hired then it retools . Eliminate choice (A). Choice (B) still has the same error. Choice (D) changes the verb form incorrectly to the future tense. Choice (E) rewrites the sentence but retains the error.

[+] See the Answer

2. Believed to be one of the first widely read female authors of the Western world, Christine de Pizan's masterwork The Book of the City of the Ladies , was written in 1405 and is a history of the Western world from the woman's point-of-view. (A) Believed to be one of the first widely read female authors of the Western world (B) Written by one of the first widely read female authors of the Western world (C) One of the first widely read female authors of the Western world, as some believe (D) Written by what some believe as one of the first widely read female authors of the Western world (E) Believed by some as one of the first works by a widely read female author in the Western world

Answer: (B) As written, this sentence has a misplaced modifier error: the book, The Book of the City of the Ladies isn't believed by anyone to be an author— Christine de Pizan is. Choices (A) and (C) repeat that error and can be eliminated. Choices (B) and (D) both change the introductory phrase to clearly refer to a written work, but choice (D) uses the incorrect idiom believe as instead of the correct form, believe to be . Choice (E) repeats that idiom error.

GMAT Critical Reasoning Questions

1. One food writer wrote that reducing the amount of animal products in one's diet can contribute to better health and well-being. Based on this claim, some people are completely eliminating meat from their diets in order to be healthier. The argument above relies on which of the following assumptions?

Answer: (B) The argument states that some people are eliminating meat from their diets because reducing the amount of animal products in one's diet can lead to better health. Meat is only one type of animal product, however. The argument assumes that by eliminating meat, the people are reducing the total amount of animal products in their diets. Choice (A) addresses increasing the amount of vegetables and grains, but the argument just deals with animal products. Choice (B) correctly addresses the people who are eliminating meat and states that those people are not increasing their consumption of dairy, which is another instance of using animal products. Thus, these people are actually reducing the amount of animal products in their diets. Choice (C) addresses most food writers, who are irrelevant to this argument. Choice (D) addresses health lifestyles, which are irrelevant to this particular argument. Choice (E) addresses the reasons behind not eating animal products, which is irrelevant to the argument.

2. Studies reveal that a daily exercise regimen helps stroke survivors regain dexterity in their extremities. Being given an exercise routine and having a consultation with a doctor about the exercise routine have been shown to be effective mechanisms to get patients to exercise daily. From the above information, which of the following statements can be reasonably inferred? (A) A stroke survivor that is given a detailed exercise plan and consults her physician about the plan will regain full dexterity in her extremities. (B) If a stroke survivor is not given an exercise plan and does not consult with a doctor, she will not regain dexterity in her extremities. (C) Stroke survivors who are given an exercise routine and consult with a doctor about that routine will sometimes regain dexterity in their extremities. (D) Being given an exercise routine and having a consultation with a doctor about the routine is the best way to help a stroke survivor regain dexterity in their extremities. (E) Only being given an exercise routine is necessary to regenerate dexterity in the extremities of seniors who have suffered a stroke.

Answer: (C) This is an inference question, so evaluate the passage and then look for an answer choice that can be reasonably inferred from the information. The passage states that a daily exercise regimen helps stroke survivors regain dexterity in their extremities and that survivors who are given an exercise routine and who have a consultation with a doctor about the routine have been shown to be effective at getting patients to exercise daily . So it can be inferred that if a survivor is given a routine and consults with a doctor, they are more likely to exercise daily, which will help them regain dexterity. Choice (A) is an example of extreme language. The phrasing will regain full dexterity is not promised in the information in the passage, as the passage only states that a routine and consultations may help a survivor exercise more. Eliminate (A). Choice (B) is also an example of extreme language. There is no way to discern from the information provided that a strong survivor would not regain dexterity without an exercise routine and a consultation, so eliminate (B). Choice (C) is a reasonable inference to make from the information in the passage so keep (C). Choice (D) also contains the extreme language best way . The information does not compare this method with any other method so eliminate (D). Choice (E) is recycled language and does not address consulting with a doctor so eliminate (E). The correct answer is (C).

GMAT Reading Comprehension Questions

Although oft-maligned in modern culture, the pigeon once stood not only for speed and reliability but also for grace and beauty. Darwin himself became a pigeon fancier after beginning to work with the humble Columbia livia , discovering them to be more fascinating than he had formerly believed. During the Victorian age, in fact, raising show pigeons was a popular hobby, with new breeds continuously arising as amateur (and not-so-amateur) ornithologists crossed animals in the hopes of creating ever more fantastic creatures. One of the most sought-after varieties was known as the Almond Tumbler, a name presumably derived from the color of the birds combined with the distinctive flight style. Over the course of many generations, this bird was so manipulated as to have a beak so small as to prevent the adult birds from feeding their offspring. And yet, it was wildly popular, drawing high prices at auctions and high prizes at competitions. How then did an animal once so well-loved come to be so loathed? As recently as World War II, the military used pigeons to carry messages but today, many people would kick a pigeon before they would feed one. Perhaps it is just a problem of population density - a lack of esteem for that which is ubiquitous. Pigeons have become our constant urban companions and, as such, have been transformed from symbols of peace, plenty, and prosperity, to representatives of disease and decay.

1. The primary purpose of this passage is to (A) convince the reader of the nobility of the pigeon, based on its history as a symbol of virtue (B) dissuade the reader from mistreating a once-majestic animal that has fallen from favor (C) rebut claims that the pigeon carries disease any more frequently than do other domestic animals (D) promote a renewal of pigeon fancying and a resurgence of breeds such as the Almond Tumbler (E) suggest that there might be more to the story of some urban wildlife than is commonly known

Answer: (E) The passage gives a brief description of the pigeon's place in recent human history and then goes on to contrast that with modern perspectives of the birds. Choice (A) goes too far—the author doesn't give any indication of believing the pigeon to be noble. Choice (B) focuses too specifically on a side comment in the second paragraph. Choice (C) also focuses too specifically on a side comments—the passage is not primarily about disease. Choice (D) is too strong—the passage isn't really promoting any specific action. Choice (E) remains neutral and informational, as does the passage.

2. The case of the Almond Tumbler is most analogous to which of the following? (A) a strain of wheat that can be grown in plentiful quantities but loses much of its nutritional value in the process (B) Arabian horses that are able to run at phenomenal speeds due to centuries of careful breeding designed to enhance those physical attributes (C) vitamins that were purported to provide all of the necessary nutrients but have since been found not to be very effective (D) the dachshund, a popular breed of dog that is nonetheless prone to severe back problems, due to weaknesses exacerbated by targeted breeding (E) the wild rock doves that are most commonly found nesting in the faces of cliffs far from human habitation

Answer: (D) The Almond Tumbler is described as a breed of pigeon that was very popular during the Victorian era. The passage also mentions that the selective breeding used to create that particular kind of bird also led to tiny beaks that kept parent birds from feeding their babies. Therefore, the best analogy would be another animal that is popular even though it has problems due to its design. Choice (A) is incorrect because it leaves out the aspect of popularity. Choice (B) is only positive and you need something that's also negative. Choice (C) is not about something that has been bred for a specific purpose, nor does it deal with popularity. Choice (D) correctly refers to a popular animal with a common health problem. Choice (E) does not refer to pigeons that have been bred by humans.

3. The passage suggests that (A) pigeons were once known for flying with celerity (B) the Almond Tumbler was the most beautiful breed of pigeon (C) Darwin was infatuated with his fancy pigeons (D) modern pigeons are dirtier than the fancy pigeons of yore (E) only scientists should breed new kinds of animals

Answer: (A) For a question this open-ended, it's usually best to check each of the answers against the passage. Choice (A) appears to match the opening line of the passage, which states that the pigeon once stood not only for speed and reliability. Choice (B) goes too far—although many Victorians seems to have loved the Tumbler, there's no evidence that it was definitively the most beautiful. Choice (C) also goes too far—the passage mentions that Darwin was fascinated by his pigeons, not that he was infatuated. Choice (D) draws an incorrect assumption—the passage comments that the common opinion has changed, not the pigeon itself. Choice (E) is not supported by the passage, which states that amateurs, as well as trained individuals, bred pigeons.

Below you'll find GMAT sample questions covering the two question types you'll encounter on the Quantitative section: Problem Solving and Data Sufficiency.

Problem Solving Questions

1. A certain company sells tea in loose leaf and bagged form, and in five flavors: Darjeeling, earl grey, chamomile, peppermint, and orange pekoe. The company packages the tea in boxes that contain either 8 ounces of tea of the same flavor and the same form, or 8 ounces of tea of 4 different flavors and the same form. If the order in which the flavors are packed does not matter, how many different types of packages are possible? (A) 12 (B) 15 (C) 20 (D) 25 (E) 30

Answer: (C) Begin by figuring out how many different ways you can package the tea in boxes that contains 8 ounces of tea, all of the same flavor. There are five flavors, each flavor can come in either loose leaf or bagged form, so 5 flavors x 2 forms = 10 different ways to package the tea in boxes that contain only one flavor each. Now find the number of different ways to package 4 different flavors of the same form per box. In this case, you must choose 4 of 5 possible flavors, and order does not matter, so the formula is 5 x 4 x 3 x 2 ⁄ 4 x 3 x 2 x 1 = 5 different ways to combine the 4 flavors. Each combination can come in either loose leaf for bagged form, so you have 2 different forms x 5 different combinations = 10 total possible ways to combine the 4 flavors in either bagged or loose-leaf form. Thus, the total number of combinations is 10 + 10 = 20 total combinations. The answer is choice (C).

2. Karen sold her house at a loss of 25 percent of the price that she originally paid for the house, and then bought another house at a price of 30 percent less than the price she originally paid for her first house. If she sold the first house for $225,000, what was her net gain, in dollars, for the two transactions? (A) $15,000 (B) $25,000 (C) $60,000 (D) $75,000 (E) $90,000

Answer: (A) If Karen sold her first house for $225,000 and at a loss of 25 percent, then 25 percent of the original price equals $225,000. 75 ⁄ 100 x = 225,000, so x, or the price she originally paid, equals $300,000. Thus, Karen lost $75,000 on the sale of her first house. If she bought a second house for a price of 30 percent less than $300,000, then the second house cost $210,000, so she gained $90,0000. $90,000 - $75,000 = $15,000, so the answer is choice (A).

Sample Data Sufficiency Questions

1. In a certain company, at least 200 people own manual transmission vehicles. If 12 percent of the people who own manual transmission vehicles also own automatic transmission vehicles, do more people own automatic transmission vehicles than own manual transmission vehicles? (1) 5 percent of the people who own an automatic transmissions vehicle also own a manual transmission vehicle. (2) 15 people own both an automatic transmission vehicle and a manual transmission vehicle. (A) Statement (1) ALONE is sufficient, but statement (2) alone is not sufficient. (B) Statement (2) ALONE is sufficient, but statement (1) alone is not sufficient. (C) BOTH statements TOGETHER are sufficient, but NEITHER statement ALONE is sufficient. (D) EACH Statement ALONE is sufficient. (E) Statements (1) and (2) TOGETHER are NOT sufficient to answer the question asked, and additional data are needed.

Answer: (A) According to statement (1), 5 percent of the people who own an automatic transmission vehicle also own a manual transmission vehicle. The question also indicates that 12 percent of the people who own a manual transmission vehicle also own an automatic transmission vehicle. Both figures relate to the total number who own both, so that means that 5 percent of the automatic transmission owners = 12 percent of the manual transmission owners. The overlap in ownership makes up a smaller percent of those who own automatic transmission vehicles, so there must be more people who own automatic transmission vehicles. Statement (1) is sufficient, so you can eliminate choices (B), (C), and (E). Statement (2) indicates that 15 people own both an automatic transmission vehicle and a manual transmission vehicle, so you know that 12 percent of the people who own a manual transmission is equal to 15 people. 12 ⁄ 100 = 15, so x = 125. Thus, there are 125 people who own a manual transmission vehicle. However, you have no further information to allow you to calculate the number of people who own automatic transmission vehicles, so statement (2) is insufficient. The answer is choice (A).

2. What is the value of x ⁄ 2 ? (1) x is 1 ⁄ 5 less than 9 ⁄ 10 (2) x is between 2 ⁄ 5 and 4 ⁄ 5 (A) Statement (1) ALONE is sufficient, but statement (2) alone is not sufficient. (B) Statement (2) ALONE is sufficient, but statement (1) alone is not sufficient. (C) BOTH statements TOGETHER are sufficient, but NEITHER statement ALONE is sufficient. (D) EACH Statement ALONE is sufficient. (E) Statements (1) and (2) TOGETHER are NOT sufficient to answer the question asked, and additional data are needed.

Answer: (A) Statement (1) allows you to find the value of x, so you can answer the question. (If x is 1 ⁄ 5 less than 9 ⁄ 10 , then 9 ⁄ 10 - 1 ⁄ 5 = x. 1 ⁄ 5 = 2 ⁄ 10 , so x equals 9 ⁄ 10 - 2 ⁄ 10 = 7 ⁄ 10 . If x equals 7 ⁄ 10 , then x ⁄ 2 = 7 ⁄ 10 divided by 2, or 7 ⁄ 20 .) Statement (1) is sufficient, so eliminate choices (B), (C), and (E). According to statement (2), x is between 2 ⁄ 5 and 4 ⁄ 5 . That means that one possible value for x is 3 ⁄ 5 , but another possible value is 7 ⁄ 10 . Statement (2) is insufficient, so the answer is choice (A).

Below you'll find examples of how you'll be asked to use a chart, graph, or table to answer questions on the Integrated Reasoning section.

Sample Integrated Reasoning Questions

Item 1: Andre is buying gifts for his office staff. He wants to spend exactly $280 and he can buy either sweatshirts, which cost $22, or baseball caps, which cost $26. In the table below, choose the number of sweatshirts and the number of baseball caps that Andre should buy.

Answer: Sweatshirts, 8; Baseball caps 4 To solve this question, systematically test out the answer choices. The equation you need to solve is 22s + 26h = 280, in which both s and h are integers and s represents the number of sweatshirts and h represents the number of baseball caps. So, start with plugging in 4 for sweatshirts and see if the number of baseball caps is an integer. 22(4) +26h = 280 h = 7.38 Since the number of baseball caps is not an integer, Andre could not have bought 4 sweatshirts. Keep trying more sweatshirts one by one until you find an answer that will you an integer value for baseball caps. 8 sweatshirts will give you 4 baseball caps.

GMAT sample question

Question 2-1 The ratio of the U.S. population in 2000 to the U.S. population in 1900 is closest to __. (A) 1 to 4 (B) 2 to 7 (C) 2 to 1 (D) 3 to 1 (E) 11 to 3

Answer: (E, 11 to 3) According to the graph, the U.S. population in 2000 was a little bit more than 275 million, and the U.S. population in 1900 was a little over 75 million. Since the question asks what the ratio is "closest to," these numbers are good enough to approximate. 275 to 75 can be reduced by 5 to get 55 to 15, which can be reduced by 5 again to get 11 to 3. Alternatively, you could reduce 275 to 75 by 25 to get this same ratio.

Question 2-2 The U.S. population in 1950 was approximately __ of the U.S. population in 1850. (A) 800% (B) 600% (C) 200% (D) 85% (E) 15%

Answer: (B, 600%) The question asks what percent the U.S. population in 1950 is of the U.S. population in 1850. To get this you need to calculate population 1950 ⁄ population 1850 x 100. Since the U.S. population in 1950 is higher, you want something that is greater than 100%. Eliminate 85% and 15%. Since the sentence says "approximate" and also since the remaining answer choices are not close to each other, you can estimate the values. According to the chart, the population in 1950 was about 150 million and the population in 1850 was about 25 million. Therefore, you need to calculate 150 ⁄ 25 x 100 = 6 x 100 = 600%.

Question 2-3 The U.S. population increased by approximately __ from 1900 to 1950. (A) 25% (B) 33% (C) 50% (D) 100% (E) 200%

Answer: (D, 100%) To get percent increase, you need to use the formula difference ⁄ original x 100. The population in 1900 was about 75 million, and the population in 1950 was about 150 million. The difference between the two figures is 75 million. Therefore, the percent increase is 75 ⁄ 75 x 100 = 100%.

Below you'll find a sample Analytical Writing Assessment (AWA) question. On the GMAT you'll have 30 minutes to write a critique of the argument.

Analysis of an Argument

The following appeared as part of a medical advertisement in a magazine.

A new medical test that allows the early detection of a particular disease will prevent the deaths of people all over the world who would otherwise die from the disease. The test has been extremely effective in allowing doctors to diagnose the disease six months to a year before it would have been spotted by conventional means.

Discuss how logically convincing you find this argument. In explaining your point of view, be sure to evaluate the line of reasoning and the use of evidence in the argument. For example, it may be necessary to consider what questionable assumptions underlie the thinking and what other explanations or counterexamples might weaken the arguments conclusion. You can also discuss what kind of evidence would strengthen or refute the argument, what changes in the argument would make it more logically persuasive, and what, if anything, would enable you to better evaluate its conclusion.

How will you score?

Take a GMAT practice test with us under the same conditions as the real thing. You'll get a personalized score report highlighting your strengths and areas of improvement.

START A FREE PRACTICE TEST

  • GMAT  

Featured Business Schools For You

Find MBA Programs Matched to Your Interests

Explore our featured business schools to find those that are looking for students like you.

Best Online MBA seal

Top Online MBA Programs

On a mission to increase your salary? Our Top 50 Online MBA ranking is based on academics, career outcomes, tech platforms, and more.

Best Career Prospects

Best Career Prospects

Find out which schools have the best track records for getting students jobs—and the highest starting salaries.

Top Entrepreneurship 2023 seal

Top Schools for Entrepreneurship

Ready to build your own business from the ground up? Check out these 50 graduate programs.

gmat problem solving pdf

Free MCAT Practice Test

I already know my score.

gmat problem solving pdf

MCAT Self-Paced 14-Day Free Trial

gmat problem solving pdf

Enrollment Advisor

1-800-2REVIEW (800-273-8439) ext. 1

1-877-LEARN-30

Mon-Fri 9AM-10PM ET

Sat-Sun 9AM-8PM ET

Student Support

1-800-2REVIEW (800-273-8439) ext. 2

Mon-Fri 9AM-9PM ET

Sat-Sun 8:30AM-5PM ET

Partnerships

  • Teach or Tutor for Us

College Readiness

International

Advertising

Affiliate/Other

  • Enrollment Terms & Conditions
  • Accessibility
  • Cigna Medical Transparency in Coverage

Register Book

Local Offices: Mon-Fri 9AM-6PM

  • SAT Subject Tests

Academic Subjects

  • Social Studies

Find the Right College

  • College Rankings
  • College Advice
  • Applying to College
  • Financial Aid

School & District Partnerships

  • Professional Development
  • Advice Articles
  • Private Tutoring
  • Mobile Apps
  • Local Offices
  • International Offices
  • Work for Us
  • Affiliate Program
  • Partner with Us
  • Advertise with Us
  • International Partnerships
  • Our Guarantees
  • Accessibility – Canada

Privacy Policy | CA Privacy Notice | Do Not Sell or Share My Personal Information | Your Opt-Out Rights | Terms of Use | Site Map

©2024 TPR Education IP Holdings, LLC. All Rights Reserved. The Princeton Review is not affiliated with Princeton University

TPR Education, LLC (doing business as “The Princeton Review”) is controlled by Primavera Holdings Limited, a firm owned by Chinese nationals with a principal place of business in Hong Kong, China.

close

Practice thousands of GMAT questions with top expert solutions.

Identify and improve upon mistakes efficiently using our Error Log.

Get the latest tips and news from our top GMAT professionals.

- it’s free and easy!

GMAT Problem Solving (PS)

Gmat club tests, gmat quantitative questions, view after sorting.

View first unread post

  • GMAT Prep Courses
  • MBA Programs
  • Build Your Study Plan
  • Best GMAT Books
  • All the GMAT Tests
  • GMAT Focus Tests
  • GMAT Verbal
  • Error Log Templates
  • GMAT Official Guide
  • Manhattan Prep
  • Target Test Prep
  • Experts' Global
  • Full Time MBA Rankings
  • Part Time MBA Rankings
  • Executive MBA Rankings
  • International MBA Rankings
  • Best App Tips
  • Best MBA Books
  • Application Reference
  • Free Profile Evaluation

Copyright © 2024 GMAT Club

GMAT ® is a registered trademark of the Graduate Management Admission Council ® (GMAC ®). GMAT Club's website has not been reviewed or endorsed by GMAC.

gmat problem solving pdf

Test-Guide.com Logo

Home > Grad School Admissions > GMAT > GMAT Practice Test

GMAT Practice Test

Exam summary.

0 of 10 Questions completed

Information

You have already completed the exam before. Hence you can not start it again.

Exam is loading…

You must sign in or sign up to start the exam.

You must first complete the following:

Exam complete. Results are being recorded.

Time has elapsed

You have reached 0 of 0 point(s), ( 0 )

Earned Point(s): 0 of 0 , ( 0 ) 0 Essay(s) Pending (Possible Point(s): 0 )

  • GMAT Data Sufficiency 0%
  • GMAT Problem Solving 0%

Congrats on taking our GMAT Sample Quiz. Take one of our full-length GMAT practice tests or one of our study sets. Everything is 100% free!

1 . Question

The school held a community service project collecting trash. Students were put into groups of 4, and each group was required to collect 15 bags of trash. The school collected 1,305 bags of trash. How many students participated?

2 . Question

Solution X is made of two ingredients, d and g , and has a ratio of 1:3. Solution Y is made of two ingredients, g and f , and has a ration of 3:2. Solution Z is made of Solution X and Y with a ratio of 4:11. If you make 40 ounces of solution Z, what percent of it will be ingredient g ?

3 . Question

The lecture hall is 35% full. A group of fourteen enter the hall and take their seats. The hall is now 42% full. How many people will still be in the hall if 17 people leave?

4 . Question

A solution of water and vinegar completely fills a 2-quart container. What percent of the solution is vinegar?

  • There is more water than vinegar in the solution.
  • The ratio of vinegar to water is 1:7.
  • a. Statement (1) ALONE is sufficient, but statement (2) alone is not sufficient to answer the question asked.
  • b. Statement (2) ALONE is sufficient, but statement (1) alone is not sufficient to answer the question asked.
  • c. BOTH statements (1) and (2) TOGETHER are sufficient to answer the question asked, but NEITHER statement ALONE is sufficient to answer the question asked.
  • d. EACH statement ALONE is sufficient to answer the question asked.
  • e. Statements (1) and (2) TOGETHER are NOT sufficient to answer the question asked, and additional data specific to the problem are needed.

5 . Question

If 4x – y = 6, what is 16 x / 2 y ?

6 . Question

When fully evaluated, what digit is in the ones place value for the following expression?

8 12 – 3 9 – 5 5

7 . Question

Consider quadrilateral ABDF. What is the area of quadrilateral ABCE, in square units?

gmat problem solving pdf

8 . Question

The integer x is positive, while the integer y is negative. Which of the following will produce a positive integer?

  • a. xy – y
  • d. x x – y

9 . Question

Alice is keeping pitching stats for her softball team. In a certain game, the ratio of strikes to balls pitched was 2:5. Which of the following could not be the total number of pitches thrown during the game?

10 . Question

Find the angle

gmat problem solving pdf

Use Our GMAT Prep (100% Free)

All gmat practice tests.

Use the GMAT practice tests listed below to study. You will find full-length exams, study sets, and flashcards.

  • Practice Exams = Timed and Full-Length Exams
  • Practice Sets = Not Timed and Smaller Sets of Questions
  • Flashcards = Important Concepts & Terms

GMAT Practice Exam #1

GMAT Practice Exam #2

GMAT Practice Exam #3

GMAT Practice Exam #4

GMAT Quant Practice Sets

GMAT Verbal Practice Sets

GMAT Data Insights Practice Sets

GMAT Quant Flashcards

GMAT Verbal Flashcards

Overview of the GMAT

The GMAT recently underwent some big changes. Starting February 1, 2024, the Traditional GMAT is being retired and switched to the GMAT Focus Exam.

The GMAT Focus includes 3 different sections. The exam is multiple-choice, computer-based, and computer-adaptive.

The 3 sections on the exam are:

  • Data Insights: 20 Questions, 45 Minutes
  • Quantitative Reasoning: 21 Questions, 45 Minutes
  • Verbal Reasoning: 23 Questions, 45 Minutes

There are 64 total questions and you will have 2 hours and 15 minutes to complete the exam.

An image showing an overview of the GMAT

For more information on the exam, visit our  GMAT test overview . You can also visit the  official site  for additional information.

Scoring of the GMAT

The GMAT is now scored on a scale of 205-805. It used to be scored on a scale of 200-800.

All 3 sections of the exam will count towards your total scaled score. Each section is scored on the same scale:

  • Data Insights: 60-90
  • Quantitative Reasoning: 60-90
  • Verbal Reasoning: 60-90

Every section is equally weighted towards your total score. Your section scores will be scaled to arrive at your total score (between 205 and 805).

To learn more about scores, read our guide to GMAT scores for more information.

Registering for the GMAT and Test Dates

The exam is given year-round (regardless of which option you choose to use when taking the exam). You will have 2 options with regards to taking the exam:

  • Online (Remotely Proctored)
  • In-Person (Testing Center)

It is a pretty straight-forward process to register and take the GMAT. You can follow these steps to register:

  • Create an Account on MBA.com
  • Decide How You Want to Take the Exam (Online or In-Person)
  • Pick Your Test Date and Time

The biggest decision you will have to make is choosing whether to take the exam online or in-person at a testing center. The option is completely up to you and is a matter of personal preference.

The standard fee to take the exam is $275 or $300 (depending on the method you choose to take the exam).

How to Use a GMAT Practice Test

Using a practice test when studying for the GMAT is a great study method. But what is the best way to use a practice exam?

We suggest the following when using our GMAT questions:

  • Initial Practice Exam: Take a practice exam in each subject area (and each question type).
  • Answer Explanations: Review all answer explanations and determine which areas give you the most trouble.
  • Focus on Problem Areas: Focus your studying on those problem areas until you are comfortable with the material.
  • Additional Practice Exams: Take another practice exam once you feel comfortable to confirm that you comprehend the material.

Keep the following in mind when using a practice exam:

  • Explanations: Our answer explanations are meant to help you better understand why an answer was incorrect. You can use the answer explanations to learn important concepts and how to solve problems quicker.
  • Study Trends: Our practice questions should help you discover trends about yourself that you may not have known. You may discover that you seriously struggle with a certain type of question. You can use these trends to study in a more efficient manner.
  • Time: The timing aspect of the exam always gives individuals trouble. Becoming more familiar with the questions and the way in which they are asked will help you get better with the time aspect.

Benefits of GMAT Practice Questions

There are many benefits to using GMAT questions when studying for your exam. Some benefits include:

  • Becoming Comfortable With the Test Format:  The GMAT, like most standardized tests, has its own unique format. As you take more and more sample tests, you will begin to see a pattern in the way the questions are written. Once the actual test day comes, you will feel comfortable and have no surprises.
  • Helping Improve Your Ability to Solve Problems:  Standardized tests measure your ability to solve problems, not just memorize information. To do well on the GMAT test you will need to have strong problem-solving capabilities. When answering questions, pay special attention to the answer explanations presented in your score report to help improve your problem-solving abilities.
  • Improving Your Pace: Each section of the exam is timed. To do well, you need to keep a steady pace. Practice taking the sample exams in a timed format to help improve your speed and decision-making.
  • Focusing Your Study Time:  One of the biggest advantages to taking a practice test is learning what you are good at and what you are weak at. You can then concentrate your study time on your weakest areas.

Frequently Asked Questions

Is the gmat multiple-choice.

Yes, the exam is multiple-choice.

Is the GMAT taken on a computer?

The exam is usually taken on a computer at an official testing center or online (remotely proctored).

In addition, the exam is computer adaptive. This means the exam gets harder as you answer more questions correctly.

How many questions are on the exam?

There will be 64 total questions. You will have 2 hours and 15 minutes to take the exam.

How many times can you take the GMAT?

You are allowed to take the GMAT once every 16 days and only 5 times total in a 12-month period.

You can take the exam a total of 8 times in your lifetime.

When is the GMAT changing?

The last date you can take the Traditional GMAT is January 31, 2024. Starting February 1, 2024, you will only be able to take the GMAT Focus Exam.

How much does the GMAT cost?

The standard fee to take the exam is $275 or $300. This depends on the method in which you are taking the exam.

What is a perfect GMAT score?

The exam is scored on a scale of 205-805. A perfect score is an 805.

gmat problem solving pdf

  • Authored By: Trevor Klee
  • Last Updated: February 9, 2024

GMAT Quantum

Boost your GMAT Quant Score

GMAT Official Practice Questions 2

Gmat official practice questions 2: video explanations.

gmat problem solving pdf

Sign Up to access the content below.

Problem Solving

Easy (32 Problems):

PS21146, PS21147, PS21150, PS21151, PS21152, PS21154, PS21156, PS21164, PS21165, PS21166, PS21169, PS21170, PS21171, PS21183, PS21184, PS21196, PS21205, PS21206, PS21229, PS21230, PS21231, PS21232, PS21233, PS21234, PS21235, PS21247, PS21248, PS21249, PS21250, PS21253, PS21255.

Medium (35 Problems):

PS21148, PS21149, PS21153, PS21155, PS21157, PS21158, PS21159, PS21160, PS21161, PS21162, PS21163, PS21167, PS21168, PS21172, PS21173, PS21174, PS21175, PS21176, PS21177, PS21178, PS21179, PS21180, PS21197, PS21198, PS21199, PS21207, PS21236, PS21237, PS21252, PS21254, PS21256, PS21264, PS21273, PS21274, PS21278 .

Hard (34 Problems):

PS20354, PS20355, PS20358, PS20362, PS20364, PS20368, PS20369, PS20370, PS20373, PS20379, PS20380, PS20386, PS20388, PS20389, PS20396, PS20401, PS20402, PS20404, PS20405, PS20406, PS20407, PS20408, PS20409, PS20410, PS20417, PS20418, PS20426, PS20428, PS20429, PS20433, PS20439, PS20477, PS20487, PS20419.

Data Sufficiency

Easy (36 Problems):

DS20593, DS21181, DS21182, DS21185, DS21186, DS21187, DS21188, DS21189, DS21190, DS21191, DS21192, DS21194, DS21200, DS21202, DS21203, DS21204, DS21208, DS21209, DS21212, DS21218, DS21219, DS21238, DS21241, DS21242, DS21243, DS21244, DS21257, DS21258, DS21260, DS21265, DS21266, DS21267, DS21268, DS21270, DS21271, DS21272.

DS20349, DS21193, DS21195, DS21201, DS21210, DS21211, DS21213, DS21214, DS21215, DS21216, DS21217, DS21220, DS21221, DS21222, DS21223, DS21224, DS21226, DS21227, DS21228, DS21239, DS21240, DS21245, DS21246, DS21259, DS21261, DS21262, DS21263, DS21269, DS21275, DS21276, DS21277, DS21279, DS21280, DS21281, DS21282.

Hard (27 Problems):

DS20356, DS20357, DS20360, DS20361, DS20378, DS20382, DS20392, DS20393, DS20395, DS20400, DS20411, DS20412, DS20413, DS20420, DS20422, DS20434, DS20436, DS20441, DS20442, DS20463, DS20481, DS20483, DS20555, DS20556, DS20557, DS20559, DS21225.

' src=

October 5, 2020 at 10:40 AM

Dear Mr Dabral,

I’m Elvian from Indonesia, I’m curious about your course then I have a little question about it,

Did your course offer all GMAT sections from Math, Verbal, Integrated Reasoning, and the Essay? If no, did you have any recommendation about the section that not covered in your course?

I’m looking forward to take your course, but I still on saving mode so I can pay it.

' src=

October 5, 2020 at 1:35 PM

The online course at GMATQuantum only provides preparation for the quantitative reasoning section of the GMAT. I don’t teach the GMAT verbal and I am not qualified enough to recommend any resources for the verbal section.

If you have any additional questions, feel free to send me a message via the Contact me form.

Leave a Reply Cancel reply

Your email address will not be published. Required fields are marked *

IMAGES

  1. How to Improve Your GMAT Integrated Reasoning Score

    gmat problem solving pdf

  2. GMAT Problem Solving Mock Test Paper 1 PDF Free Download

    gmat problem solving pdf

  3. GMAT Problem Solving

    gmat problem solving pdf

  4. GMAT Problem Solving Questions With Answers

    gmat problem solving pdf

  5. GMAT Problem Solving Guide

    gmat problem solving pdf

  6. GMAT Problem solving Fundamentals Questions

    gmat problem solving pdf

VIDEO

  1. GMATning Problem solving masalasi sof O'zbek tilida, Ipidan-ignasigacha

  2. GMAT Tutorials: GMAT Quant

  3. GMAT Problem Solving

  4. Gmat /problem solving 5

  5. GMAT Math: Solving a probability question

  6. I Need Serious Help with GMAT Quant

COMMENTS

  1. Free GMAT Practice Questions with detailed Explanations

    Our Free Practice Questions are designed to give you the thorough understanding of how to go about solving a problem that you crave. Our thorough explanations show you what to expect from each GMAT question, detailing question-specific hurdles and common traps. Thankfully, our practice questions provide a wide variety of question types spanning ...

  2. Free GMAT Practice Questions with detailed Explanations

    You have not answered any question so far. There are 50 free practice questions in our database in total, which you can answer and will improve your skills. You can answer all questions in a row (click on "All Questions") or only all questions of a particular section (click on that Section) or a single selected question (click on that Question).

  3. Free GMAT Practice Questions with detailed Explanations

    Free GMAT Practice Questions. Question 1 of 1. ID: GMAT-DSQ-1. Section: Quantitative Reasoning - Data Sufficiency. Topics: Number Properties; Highest/Greatest Common Factor (HCF/GCF); Least Common Multiple (LCM); Word Problems. Difficulty level: Challenging. ( Practice Mode: Single selected Question » Back to Overview) In a certain class, a ...

  4. PDF 12 EXCLUSIVE GMAT EXAM QUESTIONS

    Answer: D. Explanation: This question asks you to identify a claim that is made in the passage about ozone-depleting chemicals. The best answer is D. The passage, written in 1996, states that the rate of increase in amounts of most ozone-depleting chemicals reaching the atmosphere had been reduced since 1987.

  5. Free GMAT Practice Questions with detailed Explanations

    Free GMAT Practice Questions. Question 1 of 1. ID: GMAT-PSQ-3. Section: Quantitative Reasoning - Problem Solving. Topics: Number Properties; Inequalities; Absolute numbers. Difficulty level: Challenging. ( Practice Mode: Single selected Question » Back to Overview) Given that: 4m + n = 20 4 m + n = 20; and.

  6. PDF Total GMAT Math (Excerpt)

    merits the entire next chapter. Before getting to work problems, we'll take a look at a couple of others way in which the GMAT will combine rates. The rst type of problem is "average rate" or "average speed." An example might go like this: "Karen drove 100 miles at a speed of 40 miles per hour, then another 100 miles at a speed of 50 miles ...

  7. Free Sample GMAT Practice Questions

    Each problem is followed by five potential answer choices, with only one being correct. Here are three PS sample questions for you to try. In a class of 50 students, 20 play Hockey, 15 play Cricket and 11 play Football. 7 play both Hockey and Cricket, 4 play Cricket and Football and 5 play Hockey and football.

  8. GMAT Problem Solving Sample Questions

    The total number of sample Problem Solving problems available from this page is far more than 37, the total number of math questions you will see on a full Quantitative section of the GMAT. In each blog, the solutions & explanations to the sample questions are at the ends of the articles. (If the topic is less than crystal clear for you, you ...

  9. Sample GMAT Problem Solving Questions, With Answers

    GMAT Problem Solving, Sample Question #2. A certain airline's fleet consisted of 60 type A planes at the beginning of 1980. At the end of each year, starting with 1980, the airline retired 3 of the type A planes and acquired 4 new type B planes. How many years did it take before the number of type A planes left in the airline's fleet was ...

  10. How to Master GMAT Problem Solving │ mba.com

    Grab your phone and set the timer for 6 minutes. (If you've been granted 1.5x time on the GMAT, set it for 9 minutes. If you've been granted 2x time on the GMAT, set it for 12 minutes.) Do the below 3 problems under real GMAT conditions: Do them in order. Don't go back. Pick an answer before you move to the next one.

  11. 10 Top Tips for GMAT Problem Solving Questions

    The key to GMAT problem solving mastery, then, lies in mastering the fundamentals. Memorize the exponent rules. Memorize common roots and higher powers. Memorize the formulas for finding area of different shapes. Know how to find mean, median, mode, and standard deviation without blinking an eye.

  12. OFFICIAL GMAT™ EXAM STUDY PLANNER

    Check-in time: Take GMAT™ Official Practice Exam 3. Review Practice Exam 3 results and assess your progress. Increase your difficulty level. Continue to study your practice questions in the Official Guide and study the answers to learn the problem-solving methodology. Take GMAT™ Official Practice Exam 4. Pay attention to pacing throughout ...

  13. 2000+ GMAT Sample Questions: Practice Every Question Type

    The GMAT Focus Quantitative Diagnostic Tool consists of a 24-question quantitative test (12 data sufficiency questions and 12 problem-solving questions) that uses real questions from retired exams. It's computer adaptive and follows the style and format of the actual GMAT quant section.

  14. PDF GMAT Word Problems: Fundamentals (Excerpt)

    3 Problem Solving 4 4 Data Su ciency 6 5 Answer Key 8 6 Explanations 9 1. Je Sackmann GMAT Tutor www.gmathacks.com 1 Introduction This document contains nothing but GMAT Word Problems questions{100 of them, to be exact. There's nothing inherently di cult about a word problem, but very few GMAT word problems are very easy. The biggest ...

  15. Poets&Quants

    Prepare for the GMAT with these official sample GMAT Problem Solving questions, including answers and analysis by an expert tutor. Toggle navigation. Wharton | Mr. Private Equity. GMAT 730, GPA 3.2. HEC Paris | Ms. M&A Lawyer. GMAT 700, GPA 3.5. Stanford GSB | Ms. UK In the USA.

  16. Sample Questions

    Before you register for the GMAT Focus Edition, get a preview of the types of questions you'll encounter on the exam. ... You can learn more about section and question type at Exam Content. Sample Questions by Section. Quantitative Reasoning. Problem Solving Question Directions: Solve the problem and indicate the best of the answer choices given.

  17. Free GMAT Quantitative Questions

    Free GMAT Quantitative Questions - AnalystPrep. The quantitative reasoning section tests your numerical literacy and mathematical abilities, such as solving quantitative problems and interpreting graphical data. It consists of 31 multiple choice questions, which should take 62 minutes (2 minutes per question) to be completed.

  18. GMAT Sample Questions

    Below you'll find GMAT sample questions covering the two question types you'll encounter on the Quantitative section: Problem Solving and Data Sufficiency. Problem Solving Questions. 1. A certain company sells tea in loose leaf and bagged form, and in five flavors: Darjeeling, earl grey, chamomile, peppermint, and orange pekoe.

  19. PDF Table of Contents

    Introduction to GMAT Problem Solving Questions Practice Question Introduction to GMAT Data Sufficiency General Strategy for Data Sufficiency Questions. gmat.magoosh.com 2 Practice Question GMAT Math: Memory vs. Memorizing The Top Five GMAT Math Formulas The Power of Estimation for GMAT Quant

  20. GMAT Math Problem Solving : Practice Tests and Information

    Practice your math problem solving skills with our 10 tests. You shouldn't need more than three lines of working for any problem. Redraw geometry figures on your scratch pad to include the information in the question. Each test has ten questions and should take 12 minutes. Reading the explanations to the questions you get wrong will strengthen ...

  21. GMAT Quantitative (Math) Forum: Free Practice Questions, PDFs and Tips

    All GMAT Prep Problem Solving (PS) and Data Sufficiency (DS) questions. Tags: Collection of Questions, Useful Links. Go to page: 1, 2. Bunuel. 20 . 108137 ... Free 23-page PDF Guide -- Beat Properties of Numbers Questions. PGTLrowanhand. 2 . 5153 . by: bumpbot. Mon Mar 11, 2024 1:56 pm. 3

  22. Free GMAT Practice Test

    To do well on the GMAT test you will need to have strong problem-solving capabilities. When answering questions, pay special attention to the answer explanations presented in your score report to help improve your problem-solving abilities. Improving Your Pace: Each section of the exam is timed. To do well, you need to keep a steady pace.

  23. GMAT Official Guide 2021: Video explanations GMATQuantum

    This page lists the video explanations to all the quantitative problems in the GMAT Official Guide 2021. The questions listed in red, 42 in total (32 Problem Solving and 10 Data Sufficiency), were newly added to the GMAT Official Guide 2021. Color code: Red are questions new to the GMAT Official Guide 2021 Color code: Blue are questions ...

  24. GMAT Official Practice Questions 2

    GMAT Official Practice Questions 2: Video Explanations. This page lists video explanations to the 199 quantitative problems in the online GMATPrep Official Practice Questions 2 released in May 2020. Each question in the practice pool has a unique ID that can be used to locate the corresponding video explanation.